Difference between revisions of "2002 AMC 12P Problems/Problem 13"

m (Solution)
m (Solution)
Line 17: Line 17:
  
 
== Solution ==
 
== Solution ==
Note that <math>k^2_1 + k^2_2 + ... + k^2_n <= \frac{k(k+1)(2k+1)}{6}</math>.
+
Note that <math>k^2_1 + k^2_2 + ... + k^2_n \leq \frac{k(k+1)(2k+1)}{6}</math>.
  
 
== See also ==
 
== See also ==
 
{{AMC12 box|year=2002|ab=P|num-b=12|num-a=14}}
 
{{AMC12 box|year=2002|ab=P|num-b=12|num-a=14}}
 
{{MAA Notice}}
 
{{MAA Notice}}

Revision as of 20:16, 10 March 2024

Problem

What is the maximum value of $n$ for which there is a set of distinct positive integers $k_1, k_2, ... k_n$ for which

\[k^2_1 + k^2_2 + ... + k^2_n = 2002?\]

$\text{(A) }14 \qquad \text{(B) }15 \qquad \text{(C) }16 \qquad \text{(D) }17 \qquad \text{(E) }18$

Solution

Note that $k^2_1 + k^2_2 + ... + k^2_n \leq \frac{k(k+1)(2k+1)}{6}$.

See also

2002 AMC 12P (ProblemsAnswer KeyResources)
Preceded by
Problem 12
Followed by
Problem 14
1 2 3 4 5 6 7 8 9 10 11 12 13 14 15 16 17 18 19 20 21 22 23 24 25
All AMC 12 Problems and Solutions

The problems on this page are copyrighted by the Mathematical Association of America's American Mathematics Competitions. AMC logo.png